Using a Power Series to approximate a definite integral

Click For Summary
The discussion focuses on using a power series to approximate a definite integral, specifically integrating a series from 0 to 1/10. The user correctly derives the power series and sets up the integral but struggles with determining the error threshold for the approximation. The error formula discussed involves the maximum of the (n+1)-th derivative over the interval, which helps in estimating the error when truncating the series. Ultimately, the user finds that summing up to N=4 provides an acceptable error margin of less than 10^-5, using trial and error to identify this value. The conversation emphasizes understanding error estimation in series approximations.
Bazzinga
Messages
45
Reaction score
0
[PLAIN]http://img600.imageshack.us/img600/1210/11096142.png

Hey I was wondering if you guys could help me out with this question...

I think I have the right power series:

= \frac{1}{1-x} + \frac{x}{1-x}
= (1+x+x^{2}+x^{3}+...)(x+x^{2}+x^{3}+x^{4}+...)
= 1+2x+2x^{2}+2x^{3}+2x^{4}+...
= 1 + \sum^{infinity}_{n=1} 2x^{n}

Then I integrate to get:

\int^{1/10}_{0} 1 + \sum^{infinity}_{n=1} 2x^{n} dx
=x + \sum^{infinity}_{n=1} \frac{2x^{n+1}}{n+1}|^{1/10}_{0}
= \frac{1}{10} + \sum^{infinity}_{n=1} \frac{(1/5)^{n+1}}{n+1}

So I have the integral, I'm just not sure how to find an answer such that the error is less than 10-5 (I've never been any good with errors...)

We did learn in class today something where the error is less than
\frac{M}{(n+2)!} (b-a)^{n+2}

Am I supposed to rearrange this formula? And what's M?
 
Last edited by a moderator:
Physics news on Phys.org
If you keep n+1 terms in your sum and neglect the rest, this formula gives you the (maximal) error you can have. M is the maximum of the (n+1)-st derivative of your function over the interval [a,b]=[0,1/10], which is in your case given by the next term in the power series, the first one you neglected, times (n+1)!, since you use a power series and not (directly) the derivative.
Beware: you may have to change n+1 for n+2 above, I'm not quite sure there, usually one truncates a series at the termn and then in the formulae there are n+1's insteadof the n+2 you gave us ...
 
One other note: When evaluating the limits inside the Sigma, exponentiation takes precedence over multiplication by coefficients. For 2 * x^(n+1) evaluated at x = 1/10, the correct evaluation is expressed as 2*(1/10)^(n+1), so that (1/10) is raised to the power (n+1) BEFORE multiplying by 2.
 
I must have copied down the formula wrong, woops!

Im still not understanding.. I can find the error when finding an approximation for the series, but I don't understand how to find an approximation of the series when I have an error...
 
Instead of summing all terms in your result up to n=infinity, you just sum until a finite n=N. This is your approximation. Of course it isn't correct, but there comes the error formula: it tells you how big your mistake can be at most.
Therefore, you have to find a N such that the error, evaluated for that N, is smaller than 10^(-5), and use this N. (You should get N=4).
 
I ended up using trial and error and I ended up with N=4. I'm not sure if there's a better way to find M, but I made a list of derivatives and found it that way and that worked! Thanks!
 
Question: A clock's minute hand has length 4 and its hour hand has length 3. What is the distance between the tips at the moment when it is increasing most rapidly?(Putnam Exam Question) Answer: Making assumption that both the hands moves at constant angular velocities, the answer is ## \sqrt{7} .## But don't you think this assumption is somewhat doubtful and wrong?

Similar threads

  • · Replies 105 ·
4
Replies
105
Views
6K
  • · Replies 1 ·
Replies
1
Views
2K
  • · Replies 2 ·
Replies
2
Views
2K
  • · Replies 6 ·
Replies
6
Views
2K
  • · Replies 7 ·
Replies
7
Views
2K
  • · Replies 9 ·
Replies
9
Views
3K
  • · Replies 12 ·
Replies
12
Views
2K
  • · Replies 10 ·
Replies
10
Views
2K
  • · Replies 1 ·
Replies
1
Views
2K
Replies
8
Views
2K